69 of 100: No Trigonometry Required!

Geometry Level 1

The big square above is divided into nine congruent squares. What is the sum of the measures of the three shaded angles?

As usual for the geometry problems in this challenge, there are many ways to approach this problem. For example, there's one clever technique that involves shifting the segments to different positions on the grid.

15 0 150^\circ 17 5 175^\circ 18 0 180^\circ 21 0 210^\circ

This section requires Javascript.
You are seeing this because something didn't load right. We suggest you, (a) try refreshing the page, (b) enabling javascript if it is disabled on your browser and, finally, (c) loading the non-javascript version of this page . We're sorry about the hassle.

26 solutions

It's obvious that x = 45°. Mirror y vertically and try to figure out what the gap between y and z is. Looking at the grid you can see that AB = AC. Therefore in triangle ABC angle B = angle C. Also, noting that the slopes of AB and AC are negative reciprocals, you can see that angle A = 90°. Thus angle B = angle C = 45°. So x will perfectly fit into angle C. Conlusion: x + y + z = 180°.

@Zach Abueg - If you replace "looking at the grid" with "noticing that the slopes of AB and AC are negative reciprocals", then I think that you have a fully satisfying solution.

John Hagen - 3 years, 10 months ago

Log in to reply

Yes! Perfect, thank you John.

Zach Abueg - 3 years, 10 months ago

I was waiting in the hope Michel would make the observation and change it, but since this answer is by far the most popular I went ahead and changed it for him. Michel, feel free to tweak the wording if you desire.

Jason Dyer Staff - 3 years, 10 months ago

"Looking at the grid" is not enough to mathematically justify a result. Use the grid to supplement a proof, not to become one. For example, how do we know A = 9 0 \angle A = 90^{\circ} ?

Zach Abueg - 3 years, 10 months ago

Log in to reply

@Michel van den Heuvel I think "looking at the grid" is fine for AB = AC -- the portions are absolutely congruent, just rotated -- but the 90 degrees does need an argument. It doesn't have to be a complicated argument, though.

Jason Dyer Staff - 3 years, 10 months ago

Log in to reply

Fair enough, thanks Jason :)

Zach Abueg - 3 years, 10 months ago

Zach, you're back! :D anyway yes I was going to ask that, thanks! :P

Angel ONG - 3 years, 10 months ago

Log in to reply

I won't be leaving this site any time soon, my friend :)

Zach Abueg - 3 years, 10 months ago

Log in to reply

@Zach Abueg That's not what I meant... but that's cool too, I guess! :P Keep writing solutions and keep on solving puzzles, you're awesome! :)

Angel ONG - 3 years, 10 months ago

Log in to reply

@Angel Ong Oh no, I was just joking with you! Of course buddy :) And thank you!

Zach Abueg - 3 years, 10 months ago

Bacause tan a1 = 1/2 and tan a2 = 2/1, so tanA = (1/2 +2/1)/(1 - 1/2*2/1) = that wonderful theortical necessity ie infinity, so A (being less than 180, must be 90

Katherine barker - 3 years, 10 months ago

https://m.youtube.com/watch?v=m5evLoL0xwg Far better solution

Joe Aulicino - 3 years, 10 months ago

can anyone plz tell me that according to this solution how clealy is angle A=90

Shubhjeet Grover - 3 years, 10 months ago

Nice. I had to use arctan for this one. Thanks for showing this idea.

A Former Brilliant Member - 3 years, 10 months ago

This is what I did

Prathik Diwakar - 3 years, 10 months ago
Sharky Kesa
Aug 7, 2017

Solution 1: Translation and Dilation

We can rearrange and resize the angles to get:

Solution 2: Addition through tangent identities

tan 1 ( 2 ) + tan 1 ( 3 ) = x tan ( x ) = tan ( tan 1 ( 2 ) + tan 1 ( 3 ) ) = tan ( tan 1 ( 2 ) ) + tan ( tan 1 ( 3 ) ) 1 tan ( tan 1 ( 2 ) ) tan ( tan 1 ( 3 ) ) = 2 + 3 1 2 × 3 = 1 x = 13 5 tan 1 ( 1 ) = 4 5 tan 1 ( 1 ) + tan 1 ( 2 ) + tan 1 ( 3 ) = tan 1 ( 1 ) + x = 18 0 \begin{aligned} \tan^{-1}(2)+\tan^{-1}(3) &= x\\ \tan(x) &= \tan (\tan^{-1}(2)+\tan^{-1}(3))\\ &= \dfrac{\tan (\tan^{-1}(2)) + \tan (\tan^{-1}(3))}{1- \tan (\tan^{-1}(2)) \tan (\tan^{-1}(3))}\\ &= \dfrac{2+3}{1-2 \times 3}\\ &= -1\\ x &= 135^{\circ}\\ \tan^{-1} (1) &= 45^{\circ}\\ \implies \tan^{-1}(1)+\tan^{-1}(2)+\tan^{-1}(3) &= \tan^{-1} (1) + x\\ &= 180^{\circ}\\ \end{aligned}

https://m.youtube.com/watch?v=m5evLoL0xwg

Joe Aulicino - 3 years, 10 months ago
Tom Verhoeff
Aug 8, 2017

Use complex numbers : ( 1 + i ) ( 1 + 2 i ) ( 1 + 3 i ) = ( 1 + 3 i ) ( 1 + 3 i ) = 1 9 = 10 (1+i)(1+2i)(1+3i) = (-1+3i)(1+3i) = -1-9 = -10 . In complex multiplication, the argument angles get added. The argument angle of 10 -10 is \fbox{\$180^{\circ}\$} .

Even if this solution is correct - actually being the same I've used to prove myself my first impression was correct - I think it can be considered a solution using trigonometry, but the title explicitly said that no trigonometry was required.

Carlo Ferretti - 3 years, 9 months ago
Atomsky Jahid
Aug 7, 2017

tan 1 3 + tan 1 2 + tan 1 1 = π = 18 0 \tan^{-1}3+\tan^{-1}2+\tan^{-1}1=\pi=180^{\circ}

Proof: tan 1 3 + tan 1 2 + tan 1 1 = tan 1 3 + 2 1 3 × 2 + tan 1 1 \tan^{-1}3+\tan^{-1}2+\tan^{-1}1=\tan^{-1}\frac{3+2}{1-3\times 2}+\tan^{-1}1 = tan 1 ( 1 ) + tan 1 1 =\tan^{-1}(-1)+\tan^{-1}1 = tan 1 1 + 1 1 + 1 =\tan^{-1}\frac{-1+1}{1+1} = tan 1 0 =\tan^{-1}0 = π =\pi In the last step, we took the value of tan 1 0 \tan^{-1}0 to be π \pi as the summation of those three anges must be greater than zero.

Favourite solution, just not immediately obvious that the arctan 2 labelled is true:)

Dan Ley - 3 years, 10 months ago

But the title was "No Trigonometry required" ?

Robert DeLisle - 3 years, 10 months ago

Log in to reply

I didn't see the title at first. That's why I also added a trigonometric solution.

Atomsky Jahid - 3 years, 10 months ago

Log in to reply

I didn't notice it either, and forgot I had already written it up in my math puzzle picture album, saw the three tangents 1,2,3 right off, hit the calculator ATAN to get 180 in under a minute. Then I worked out the pure geometric solution again, but fortunately remembered that it was already in my FB album ready to repost here before starting another .jpg version.

Robert DeLisle - 3 years, 10 months ago

Log in to reply

@Robert DeLisle This happens to me also. I can rarely remember the problems I solve.

Atomsky Jahid - 3 years, 10 months ago

Log in to reply

@Atomsky Jahid I still wasn't happy with the old one. Too busy. It has been replaced with a cleaner version in my solution post.

Robert DeLisle - 3 years, 10 months ago

I don't think the trigonometry is relevant, the ratio of sides is the same as the original so only the total angle matters.

Dan Ley - 3 years, 10 months ago

I got 45 for angle 1 67.5 for angle 2 and 75 degrees for angle 3

187.5 total

Mathew Rizkalla - 3 years, 10 months ago

https://m.youtube.com/watch?v=m5evLoL0xwg

Joe Aulicino - 3 years, 10 months ago
Mourits de Beer
Aug 8, 2017

This is question # 69 69 , and it is clear that the number 6 6 is 9 9 rotated 18 0 180^\circ . Mathematicians have a good sense of humour \Rightarrow the answer is 18 0 \boxed{180^\circ} .

Robert DeLisle
Aug 8, 2017

Tea Mill
Aug 8, 2017

Quite elegant :)

Kazem Sepehrinia - 3 years, 10 months ago

I rearranged the shaded angles and voila! A bit wonky, but nevertheless, a straight line which (as we all know) is 180 degrees.

AnnaMarie Hauser
Aug 8, 2017

I knew x = 45 degrees because it would be exactly half of 90 degrees. Then, even though we didn't have to use trigonometry, I used a inverse trig functions (and a calculator) to find tan-1 (1/2) to find part of what was missing in triangle y. Tan-1 (1/2) is about 27 degrees, so < y would be 180 - 90 - 27 or 63 degrees. Then I used the same process to find < z. Tan-1 (1/3) is about 18 degrees, so < z = 180 - 90 - 18 or 72 degrees. Lastly I added 45 + 72 + 63 to get 180 degrees.

Joules Jams
Aug 8, 2017

x is the argument of the complex number 1+i

y is the argument of 1+2i

And z is the argument of 1+3i

When complex numbers are multipled together there arguments add together

(1+i) (1+2i)(1+3i) = (-1+3i) (1+3i) = -10

The argument of -10 is 180° so x+y+z=180° QED

Daniele Grandini
Aug 29, 2017

Draw the square whose vertices are ( 0 , 1 ) (0,1) , ( 1 , 3 ) (1,3) , ( 3 , 2 ) (3,2) and ( 2 , 0 ) (2,0) (grid coordinates). The angle formed by one of the sides and the diagonal is 4 5 = z ( 9 0 y ) , 45^{\circ}=z-(90^{\circ}-y), whence z + y = 13 5 z+y=135^{\circ} and x + y + z = 18 0 x+y+z=180^{\circ} .

Julie Hindley
Aug 17, 2017

atan(1)+atan(2)+atan(3)=180

Jet Sri
Aug 16, 2017

The line segments are rearranged and extended to form three sides of a triangle. In the process the three shaded angles have become the angles of a triangle. And we know that the sum of all the angles of a triangle is 180 ° 180° .

Michael King
Aug 10, 2017

Adjacent leg of each angle is 1; opposite of x = 1; opposite of y = 2; opposite of z = 3. Applying soh-cah-TOA, we need only find the arc-tangent of each quotient:
1 tan ^-1 = 45, 2 tan ^-1 = 63.43494882, 3 tan ^-1 = 71.56505118. Adding the three arc-tangents together, we get 180 degrees.

Akshay Gupta
Aug 9, 2017

x = 45 { 45 }^{ \circ }

y = t a n 1 ( 2 ) { tan }^{ -1 }(2)

z = t a n 1 ( 3 ) { tan }^{ -1 }(3)

y + z = t a n 1 ( 2 + 3 1 2 3 ) { tan }^{ -1 }(\frac{2 + 3}{1 - 2*3})

= t a n 1 ( 5 5 ) { tan }^{ -1 }(\frac{5}{-5})

= t a n 1 ( 1 ) { tan }^{ -1 }(-1)

= 135 { 135 }^{ \circ }

x + y + z = 45 + 135

= 180 { 180 }^{ \circ }

谦艺 伍
Aug 8, 2017

x = 4 5 \angle x = 45^{\circ}

angle between pink line and blue line + angle between pink line and green line = 4 5 45^{\circ}

x + y + z = 3 x + 4 5 = 18 0 \angle x + \angle y + \angle z = 3\angle x + 45^{\circ} = 180^{\circ}

Dan Lawson
Aug 8, 2017

Look at Michel Van den Heuvel's solution. Good solution. I'd like to prove this somewhat rigouriously using no visuals but the same basic idea. (Trigonometric proof)

Without the picture, we would just have to prove that 1) triangle ABC has the same ratio of cathethi as the x-triangle i.e. that tan ( C ) = 1 \tan(\angle C)=1 , and that

2) the triangle seen with those side lengths is actually right.

Name the given right triangles z-triangle, y-triangle and x-triangle respecticely.

1) This first result ist easily obtained, when you identify the construction of triangle ABC based on its hypotenuse and one cathesus. Inspired by the picture we carry on:

The pythagorean theorem provides that a right triangle with cathethis equaling the hypotenuse of the y triangle and hypotenuse equaling the cathehi of the y-triangle (that is ABC, right?) is of ratio 1.

These are 5 \sqrt{5} and ( 10 ) \sqrt(10) , And the resulting last cathethus is ( 10 5 ) = ( 5 ) \sqrt(10-5) = \sqrt(5)

2) To prove that ABC is right, I'll show that the right triangle constructed on the hypotenuse of the x-triangle and with hypotenuse along the z-triangles hypotenuse necessarily has to have its hypotenuse equaling the hypotenuse of the z-triangle. So for this right triangle we have

cos ( 180 z y ) = cos ( 90 v ) \cos(180-z-y) = \cos(90-v) for some v ( 0 ° , 90 ° ) v \in (0°,90°) (because the triangle is right, right?)

Carrying on with the cosine relation cos ( 90 v ) = a d j a c e n t h y p o t e n u s e = 5 h y p \cos(90-v) = \dfrac{ adjacent }{hypotenuse} = \dfrac{√5}{hyp}

Now since the hypothenuse of the triangle is along the z-triangle, we have

h y p = k 10 hyp = k \cdot √10 for some real k.

Insert into the prior relation to get

cos ( 90 v ) = 5 k 10 \cos(90-v) = \dfrac{√5}{k\cdot √10}

\Updownarrow

cos ( 90 v ) k 10 = 5 \cos(90-v) \cdot k√10= √5

\Updownarrow

cos ( 90 v ) k = 1 2 \cos(90-v) \cdot k = \dfrac{1}{√2}

And thus we get that h y p = 1 2 10 = 10 2 = 5 hyp = \sqrt{\frac{1}{2}}\sqrt{10}=\sqrt{\dfrac{10}{2}}=√5

And we are done. We've shown that given ABC is a right triangle with those side lengths, it has the same ratio as the right x-triangle and thus the same angles, and also that a right triangle with the given hypotenuse of √5 must be exactly ABC.

Kyle Smith
Aug 8, 2017

X is 45, and something from geometry 10 years ago stuck because I knew y is 60 and z is 75. No complex math just something about the 3x3 grid.

Except the angles are not 60 and 75. Plug 2, and 3 into the ATAN on your calculator to get the right values.

Robert DeLisle - 3 years, 10 months ago
Travis Fox
Aug 8, 2017

Each angle is initially given as a vector with respect to the Cartesian plane. First find the unit vector for x, (rt2/2, rt2/2). Then convert the plane to these units and use the new plane to rotate to the vector (1, 2) in terms of these units. Normalize this to a unit vector and repeat the process with the following plane.

Dan Cannon
Aug 8, 2017

The problem statement "No trigonometry required" and the solution choices make it a giveaway. 180 degrees is the only possible solution option that could be solved for without trigonometry.

Tom Engelsman
Aug 8, 2017

The required angle sum can be expressed as x = a r c t a n ( 1 ) + a r c t a n ( 2 ) + a r c t a n ( 3 ) a r c t a n ( 1 ) + a r c t a n ( 2 ) = x a r c t a n ( 3 ) . x = arctan(1) + arctan(2) + arctan(3) \Rightarrow arctan(1) + arctan(2) = x - arctan(3). . If we take the tangent of both sides, then we obtain:

t a n [ a r c t a n ( 1 ) + a r c t a n ( 2 ) ] = t a n [ x a r c t a n ( 3 ) ] ; tan[arctan(1) + arctan(2)] = tan[x - arctan(3)];

or t a n ( a r c t a n ( 1 ) ) + t a n ( a r c t a n ( 2 ) ) 1 t a n ( a r c t a n ( 1 ) ) t a n ( a r c t a n ( 2 ) ) = t a n ( x ) t a n ( a r c t a n ( 3 ) ) 1 + t a n ( x ) t a n ( a r c t a n ( 3 ) ) ; \frac{tan(arctan(1)) + tan(arctan(2))}{1 - tan(arctan(1)) tan(arctan(2))} = \frac{tan(x) - tan(arctan(3))}{1 + tan(x) tan(arctan(3))};

or 1 + 2 1 2 = t a n ( x ) 3 1 + 3 t a n ( x ) ; \frac{1 + 2}{1 - 2} = \frac{tan(x) - 3}{1 + 3tan(x)};

or 3 9 t a n ( x ) = t a n ( x ) 3 ; -3 - 9tan(x) = tan(x) - 3;

or 10 t a n ( x ) = 0 t a n ( x ) = 0 x = a r c t a n ( 0 ) . 10tan(x) = 0 \Rightarrow tan(x) = 0 \Rightarrow x = arctan(0).

Of the available choices, x = 180 x = \boxed{180} degrees works.

Nathalie Weibel
Aug 8, 2017

x is the argument of the complex number (1+i), y that of (1+2i) and z that of (1+3i).

x+y+z is the argument of (1+i)(1+2i)(1+3i) = -10

Therefore x+y+z = 180 °

Syrous Marivani
Aug 8, 2017

tan y = 2, tan z = 3, tan(y + z) = (tan y + tan z)/(1 – tan y tan z), tan(y + z) = -1, y + z = 135 degrees. Therefore x + t + z = 45 + 135

= 180 degrees.

John Allums
Aug 8, 2017

I know this isn't a solution from myself, but I thought some might enjoy this relevant video.

Numberphile Video - The Three Squares Geometry Problem

For what's it's worth, my solution to the problem was the one that used the $tan(A+B)$ formula.

Tangent (y+z) is equal to -1 (by tangent sum identity)

So (y+z) is equal to 135 degrees

0 pending reports

×

Problem Loading...

Note Loading...

Set Loading...